what is the magnitude of the impulse on an 7.2- kg ball rolling at 2.4 m/s when it bumps into a pillow and stops?

Answers

Answer 1

The magnitude of the impulse on a 7.2-kg ball rolling at 2.4 m/s when it bumps into a pillow and stops can be calculated using the impulse-momentum theorem.

According to the theorem, the impulse of an object is equal to the change in momentum of the object. Since the ball has a mass of 7.2 kg and an initial velocity of 2.4 m/s, its initial momentum is 17.28 kg·m/s. As the ball stops when it hits the pillow, its final momentum is 0. The change in momentum is therefore -17.28 kg·m/s.

Since the impulse of an object is equal to its change in momentum, the impulse of the ball is -17.28 kg·m/s. This means that the impulse of the ball is equal to the magnitude of the force applied to the ball multiplied by the duration of the collision. Thus, the magnitude of the impulse on the ball when it bumps into the pillow and stops is -17.28 kg·m/s.

Know more about impulse-momentum here

https://brainly.com/question/11674698#

#SPJ11


Related Questions

Suppose you were to compare three stars with the same surface temperature. If star A is a giant star, star B is a supergiant star, and star C is a main sequence star, order the three stars in terms of increasing radius. a. Star C, Star A, Star B b. Star B, Star A, Star C c. Star A, Star C, Star B d. Star B, Star C, Star A

Answers

If star A is a giant star, star B is a supergiant star, and star C is a main sequence star, the order of the three stars in terms of increasing radius is Star A, Star C, Star B.

A giant star is a luminous star that is considerably larger and brighter than the sun. The distinction between giant and dwarf stars is primarily determined by their luminosity, and giant stars are more luminous. They are not, however, larger in diameter than dwarf stars. Their size is the outcome of a high luminosity-to-mass ratio.

A supergiant star is a massive star with a luminosity that is many times greater than that of a giant star. As a result, a supergiant star is much larger than a giant star. However, supergiant stars have a similar surface temperature as giant stars.

Sequence stars are stars that spend most of their lives in the primary sequence of stars. A main-sequence star is a star that is in the hydrogen-burning phase of its evolution. It is in a state of hydrostatic equilibrium, meaning that the gravitational force holding the star together is balanced by the pressure generated by the thermonuclear fusion taking place in its core.

The stars will have the following order in terms of increasing radius: Star A, Star C, Star B if star A is a giant star, star B is a supergiant star, and star C is a main sequence star, and they all have the same surface temperature.

To learn more about surface temperature of stars https://brainly.com/question/31116168

#SPJ11

Fill in the blank:the equilibrium constant, k, relates quantity of products to reactants at a point when the reaction is ____.

Answers

The equilibrium constant, k, relates the quantity of products to reactants at a point when the reaction is at equilibrium.

What is equilibrium?

Equilibrium is a state in which the rate of the forward reaction is equal to the rate of the reverse reaction, resulting in the concentration of the reactants and products remaining unchanged. A reaction is said to be in equilibrium when it has reached a state of dynamic balance.

The equilibrium constant (Kc) is a measure of the extent to which a reaction proceeds to form products. The equilibrium constant is a ratio of the concentration of the products to the concentration of the reactants at equilibrium. The value of Kc varies with temperature and depends on the stoichiometry of the balanced chemical equation.

The larger the value of Kc, the greater the concentration of products relative to reactants at equilibrium. Similarly, a smaller value of Kc indicates a greater concentration of reactants at equilibrium. The equilibrium constant is useful in predicting the direction in which a chemical reaction will proceed.

If the value of Kc is greater than one, the equilibrium favors the products, and if the value of Kc is less than one, the equilibrium favors the reactants. If the value of Kc is equal to one, the reaction is said to be at equilibrium, and the concentration of the reactants and products is equal.

To know more about equilibrium refer here:

https://brainly.com/question/30694482#

#SPJ11

mercury's average density is about 1.5 times greater than that of earth's moon, even though the two bodies have similar radii. what does this suggest about mercury's composition?

Answers

Mercury's average density is about 1.5 times greater than that of Earth's moon, even though the two bodies have similar radii. This suggests that Mercury's composition is denser than that of Earth's moon.

The density of a substance is defined as the ratio of its mass to its volume. Mercury's average density is about 5.427 grams per cubic centimeter (g/cm³), whereas the average density of Earth's moon is about 3.34 g/cm³. Despite the fact that Mercury and Earth's moon have similar radii, Mercury's density is approximately 1.5 times greater than that of Earth's moon, indicating that Mercury's composition is denser than that of Earth's moon.

Mercury, unlike the Moon, has a large iron core, which contributes to its high density. The high density of Mercury's core, which is thought to account for about 60% of the planet's mass, is caused by the fact that it is composed primarily of iron and nickel.

Learn more about density at https://brainly.com/question/1354972

#SPJ11

if you have 7 total 100-w light bulbs in a parallel circuit in your basement and you leave them on for 1.5 days, how much energy (in kilowatt hours) would be used?

Answers

The energy consumed by the 7 100-watt light bulbs left on for 1.5 days is 25.2 kWh.

Given:

Total bulbs = 7

Power of each bulb = 100 W

Time = 1.5 days

To find: Energy used in KWh; Formula used: Energy = Power * Time

Energy used by one bulb in a day = 100 W * 24 hours = 2400 Wh = 2.4 KWh

Total energy used by one bulb in 1.5 days = 2.4 KWh * 1.5 = 3.6 KWh

Total energy used by 7 bulbs in 1.5 days = 3.6 KWh * 7 = 25.2 KWh

Therefore, 25.2 KWh of energy would be used by 7 total 100-w light bulbs in a parallel circuit in your basement and you leave them on for 1.5 days.

To know more about parallel circuits click here:

https://brainly.com/question/11409042

#SPJ11

what could the maxwell equation below be used for? select the correct answer select this answer if none of the choices are valid your answer to predict the electric field in a region of space containing many charged particles to predict what currents need to flow through wires to produce a certain electric field to predict the magnetic field in a region of space in which the electric flux is changing to predict the magnetic flux through a closed surface

Answers

The Maxwell equation ∇ × E = -∂B/∂t can be used to predict the magnetic field in a region of space in which the electric flux is changing.

The Maxwell equation ∇ × E = -∂B/∂t is one of the four Maxwell equations that describe the behavior of electric and magnetic fields. It relates the curl of the electric field to the time rate of change of the magnetic field. In other words, it describes how a changing electric field creates a magnetic field.

This equation is important in the study of electromagnetic waves, which are generated by changing electric and magnetic fields. When an electric field changes in time, it creates a magnetic field, which then creates an electric field, and so on, creating a self-sustaining wave.

The equation can be used to predict the behavior of electromagnetic waves in space, as well as the behavior of electric and magnetic fields in the presence of each other.

To know more about Maxwell equation, refer here:

https://brainly.com/question/28956380#

#SPJ11

g what is the expected acceleration of the cart if its mass is 220 g and two masses of 250 g each are added to it. a mass of 60 g is placed in the mass hanger of 5 g.

Answers

The expected acceleration of the cart when two masses of 250 g are added to it, and a mass of 60 g is placed in the mass hanger of 5 g, given that its mass is 220 g is 1.55 m/s².

What is acceleration?

Acceleration is the change in velocity with respect to time. It can be defined as the rate at which the velocity of a body changes with respect to time. It is denoted by "a".

Mass is the amount of matter in a body or object. It is a scalar quantity, which is denoted by "m".

acceleration: a = (v - u) / t

Where; a = acceleration

v = final velocity

u = initial velocity

t = time taken

The expected acceleration of the cart:

Mass = Mass of cart + Mass of 2 masses

Mass = 220 g + (2 × 250 g)

Mass = 720 g

The total mass hanging on the mass hanger:

Mass on hanger = Mass of hanger + Mass on a hanger

Mass on hanger = 5 g + 60 g

Mass on hanger = 65 g

The net force acting on the system.

Net force = (Mass on hanger + Mass) × gNet force

                = (65 g + 720 g) × 9.8 m/s²

Net force = 7.06 N

The expected acceleration of the cart;

a = F / ma

  = (7.06 N) / (720 g)

a = (7.06 N) / (0.72 kg)a

  = 9.81 m/s² × (7.06 / 0.72)a

  = 1.55 m/s²

Therefore, the expected acceleration of the cart when two masses of 250 g are added to it, and a mass of 60 g is placed in the mass hanger of 5 g, given that its mass is 220 g is 1.55 m/s².

To know more about velocity:

https://brainly.com/question/29519833

#SPJ11

what is the speed acquired by a freely falling object 5 s after being dropped from a rest position? what is the speed 6 s after?

Answers

The speed acquired by the body is 49m/s and 59m/s respectively.

The speed can be calculated using the formula:

v= u + gt,  where v= final speed, u= initial speed = 0 for a freely falling body, g= acceleration due to gravity, t= time.

The speed acquired by a freely falling object 5 seconds after being dropped from a rest position is 49 m/s. This is because an object dropped from rest will accelerate at a rate of 9.8 m/s², so after 5 seconds it will be moving at a speed of 5 * 9.8 = 49 m/s.

The speed 6 seconds after being dropped from a rest position is approximately 59 m/s. This is because an object dropped from rest will accelerate at a rate of 9.8 m/s², so after 6 seconds it will be moving at a speed of 6 * 9.8 = 58.8 m/s.


In summary, the speed of an object dropped from rest 5 seconds after being dropped is 49 m/s, and 6 seconds after it is approximately 59 m/s.

To know more about speed, refer here:

https://brainly.com/question/17661499#

#SPJ11

what is the relationship between weight and best range airspeed (vbr) and best endurance airspeed (vbe)?

Answers

The relationship between weight and best range airspeed (VBR) and best endurance airspeed (VBE) is that both VBR and VBE increase with an increase in weight.

What is best range airspeed (VBR)? Best range airspeed (VBR) refers to the airspeed at which an aircraft can cover the maximum possible distance with minimum fuel consumption. At this airspeed, the lift-to-drag ratio is the highest.

What is best endurance airspeed (VBE)? Best endurance airspeed (VBE) refers to the airspeed at which an aircraft can remain in the air for the longest possible time with minimum fuel consumption. At this airspeed, the lift-to-drag ratio is the highest.

Relationship between weight and VBR and VBE is that both VBR and VBE increase with an increase in weight.

An increase in weight means an increase in the required lift to keep the aircraft in the air. As a result, the airspeed at which the lift-to-drag ratio is the highest increases.

This is why both VBR and VBE increase with an increase in weight.

To know more about airspeed, refer here:

https://brainly.com/question/29597908#

#SPJ11

an electron and a proton are each placed at rest in a uniform electric field of magnitude 498 n/c. calculate the speed of each particle 44.4 ns after being released.

Answers

An electron and a proton are placed at rest in a uniform electric field of magnitude 498 N/C. The speed of electron and proton 44.4 ns after being released is -3.87 × 10⁶ m/s and 2.13 × 10³ m/s respectively.

Given data:

Electric field (E) = 498 N/C,

Time (t) = 44.4 ns = 44.4 × 10⁻⁹ s,

Mass of electron (m₁) = 9.11 × 10⁻³¹ kg,

Mass of proton (m₂) = 1.67 × 10⁻²⁷ kg.

Formula:

The acceleration produced in the electric field is given by a = qE/m, where q is the charge of the particle, E is the electric field strength, and m is the mass of the particle.

From the above formula, we can find the acceleration produced by the electric field on the electron and proton as follows:

For electron (q = -e, where e is the charge of an electron)

a₁ = qE/m₁ = -eE/m₁

= -1.6 × 10⁻¹⁹ × 498/9.11 × 10⁻³¹

= -8.73 × 10¹⁴ m/s²

For proton (q = +e, where e is the charge of an electron)

a₂ = qE/m₂ = eE/m₂

= 1.6 × 10⁻¹⁹ × 498/1.67 × 10⁻²⁷

= 4.80 × 10⁷ m/s²

Using the kinematic equation, v = u + at, where u is the initial velocity, we can find the speed of each particle 44.4 ns after being released as follows:

For electron,

v₁ = u₁ + a₁t = 0 + (-8.73 × 10¹⁴) × 44.4 × 10⁻⁹

= -3.87 × 10⁶ m/s

For proton,

v₂ = u₂ + a₂t = 0 + (4.80 × 10⁷) × 44.4 × 10⁻⁹

= 2.13 × 10³ m/s

Thus, the speed of the electron is -3.87 × 10⁶ m/s and the speed of the proton is 2.13 × 10³ m/s.

To know more about electric field, refer here:

https://brainly.com/question/15800304#

#SPJ4

An automobile has a vertical radio antenna 1.20 m long. The automobile travels at 65.0 km/h on a horizontal road where Earth's magnetic field is 50.0 μT, directed toward the north and downward at an angle of 65.0∘ below the horizontal.(a) Specify the direction the automobile should move so as to generate the maximum motional emf in the antenna, with the top of the antenna positive relative to the bottom.

Answers

The direction the automobile should move to generate the maximum motional emf in the antenna, with the top of the antenna positive relative to the bottom towards the east.

A magnetic field is an area surrounding a magnet or an electric current, characterized by the presence of a force that can attract or repel other magnetic materials. The concept of magnetic fields is significant in a variety of contexts, including electromagnetism, particle physics, and ferromagnetism.

According to Faraday's Law of Electromagnetic Induction, the emf generated in a conducting wire moving in a magnetic field is proportional to the strength of the magnetic field and the velocity of the conductor.

The magnitude of the emf is given by ε = Blv sinθ, where

- ε is the magnitude of the induced emf,

- B is the magnetic field strength,

- l is the length of the wire in the magnetic field,

- v is the speed of the conductor relative to the magnetic field, and

- θ is the angle between the velocity vector and the magnetic field vector.

Due to the given conditions in the question, we can use the above formula for calculating the maximum emf. To generate the maximum motional emf in the antenna, the automobile should move in a direction perpendicular to both the antenna and the Earth's magnetic field. The angle between the velocity vector and the magnetic field vector should be 90°.

1: Identify the direction of the magnetic field. In this case, the magnetic field is directed toward the north and downward at an angle of 65.0° below the horizontal.

2: Determine the direction perpendicular to both the antenna and the magnetic field. This can be done by using the right-hand rule. Point your right thumb in the direction of the magnetic field (north and downward at 65.0° below the horizontal) and your right index finger in the direction of the antenna (vertical). Your right middle finger will then point in the direction of the motion required to generate the maximum emf (perpendicular to both the magnetic field and the antenna).

The direction the automobile should move to generate the maximum motional emf in the antenna, with the top of the antenna positive relative to the bottom, is to the east.

Learn more about emf here:

https://brainly.com/question/17329842

#SPJ11

what is the magnitude of force required to stop a 4 000-kg car initially traveling at 10 m/s in 20.0 s

Answers

The magnitude of force required to stop a 4000-kg car initially traveling at 10 m/s in 20.0 s is 2,00 N.

The magnitude of force is mass into acceleration.

But we know that acceleration is velocity into time.

Therefore force =(mass*velocity)/time

In this problem, the car has a mass of 4,000 kg and is initially traveling at a velocity of 10 m/s.

The car comes to a stop, so the change in velocity is equal to the initial velocity (10 m/s). The time taken to stop the car is 20.0 seconds.

Substituting these values into the formula, we get:

force = (4,000 kg *10 m/s) / 20.0 s

Simplifying this expression, we get:

force = 200 N

Therefore, the magnitude of force required to stop a 4,000-kg car initially traveling at 10 m/s in 20.0 s is 200 N.

To practice more questions about magnitude of force:

https://brainly.com/question/8367599

#SPJ11

a flashlight bulb carries a current of 0.32 a for 82 s . part a how much charge flows through the bulb in this time?

Answers

Answer:

Q = I t        definition of current

Q = .32 Coul/sec * 82 sec = 26.2 coul

a sphere of radius r has charge q. the electric field strength at distance . what is the ratio of the final to initial electric field strengths if (a) q is halved, (b) r is halved, and (c) r is halved (but is )? each part changes only one quantity; the other quantities have their initial values.

Answers

A decrease in charge, q, will result in a decrease in the electric field strength. The ratio of the final to initial field strengths can be expressed as qf/qi. If q is halved, the ratio would be 0.5. If the radius, r, is halved, the ratio would be 1/2r2i/r2i, which is equal to 0.25. If r is halved, but the distance remains the same, the ratio would be 1/2r2i/r2i, which is equal to 0.25.

The electric field strength is inversely proportional to the distance from the charge, and directly proportional to the charge and the radius of the sphere. Therefore, halving the charge or radius will result in a decrease in the electric field strength. Halving the radius, with the distance remaining the same, will result in the same ratio as halving the charge because the distance will be the same in both cases.

Know more about electric field here

https://brainly.com/question/8971780#

#SPJ11

if an object is raised twice as high, its potential energy will be four times as much. half as much twice as much. impossible to determine unless the time is given.

Answers

If an object is raised twice as high, its potential energy will be four times as much.

Potential energy Gravitational potential energy According to the question, if an object is raised twice as high, its potential energy will be four times as much.

The potential energy is the stored energy of an object. It depends on an object’s position or configuration.

Potential energy is classified into three types: elastic potential energy, gravitational potential energy, and electric potential energy.

The gravitational potential energy of an object is the energy stored in an object when it is moved against the gravitational force. It depends on the mass of an object, the acceleration due to gravity, and the height an object is above the ground.

The equation for gravitational potential energy is:

GPE = mgh where GPE is gravitational potential energy in joules (J)m is the mass of the object in kilograms (kg)g is the acceleration due to gravity in meters per second squared (m/s²)h is the height of the object in meters (m).

To know more about Potential energy refer here:

https://brainly.com/question/12563191#

#SPJ11

suppose the air in a spherical baloon is being let out at a constant rate of 370 /. what is the rate of change of the radius of the balloon when the r

Answers

When the radius of a spherical balloon is 10 cm and the air is being let out at a constant rate of 370 cm3/s, the rate of change of the radius of the balloon is:  37/400π cm/s

We are supposed to find the rate of change of the radius of the balloon when the radius of a spherical balloon is 10 cm and the air is being let out at a constant rate of 370 cm3/s. This is a problem involving a balloon, air and its volume.

Let's first use the formula for the volume of a sphere to get the relationship between the volume and the radius of the spherical balloon.
V= (4/3)πr3
When differentiating both sides of the above equation with respect to time, t, we have;V= (4/3)πr3,  dV/dt= 4πr² dr/dt

From the problem, we have the radius, r = 10 cm and the rate of change of volume, dV/dt = - 370 cm³/s (since the air is being let out of the balloon).

Now we can substitute the given values into the equation to obtain;
dV/dt= 4πr²
dr/dt-370 = 4π(10²)dr/dt
dr/dt = - 370/ (4π(10²))= - 37/400π cm/s
Therefore, the rate of change of the radius of the balloon when the radius of a spherical balloon is 10 cm and the air is being let out at a constant rate of 370 cm3/s is - 37/400π cm/s.

To learn more about "Radius" here:

https://brainly.com/question/14928411#

#SPJ11

The previous question is incomplete, therefore, a properly phrased question is provided below.

What is the rate of change of the radius of a spherical balloon with a radius of 10 cm, when the air is being let out of the balloon at a constant rate of 370 cm³/s?

consider an electron gun with an accelerating potential of 288 v. what is the wavelength of the emitted electrons? enter the wavelength in nm unit

Answers

The wavelength of the emitted electrons is: equal to 1.83915 × 10^-9 m, or 1.83915 nm.

The wavelength of electrons emitted from an electron gun with an accelerating potential of 288 V can be calculated using the following equation:


λ = h/√(2mV)
where h is Planck's constant (6.62607 × 10^-34 m2 kg/s), m is the mass of the electron (9.10938 × 10^-31 kg) and V is the accelerating potential of the electron gun (288 V).

Therefore, the wavelength of the emitted electrons is equal to 1.83915 × 10^-9 m, or 1.83915 nm. This calculation is based on the de Broglie equation, which states that matter, including electrons, has wave-like properties and can be described using a wave equation.

According to the de Broglie equation, particles with a given mass and velocity have a wavelength that can be calculated by dividing Planck's constant (h) by the square root of twice the mass of the particle multiplied by its velocity.  This equation is commonly used to calculate the wavelength of electrons emitted from an electron gun.

To know more about wavelength refer here:

https://brainly.com/question/13533093#

#SPJ11

what is the component vr of velocty vector v along the radial direction from the radar gun to the car

Answers

The component vr of velocity vector v along the radial direction from the radar gun to the car is the component of the velocity that is in the direction of the radial line that connects the radar gun to the car.

It can be calculated by taking the dot product of the velocity vector and the unit vector of the radial line.

The unit vector of the radial line is a vector that has a magnitude of one and that is pointing in the direction of the radial line.

The dot product of two vectors is equal to the magnitude of the first vector multiplied by the projection of the second vector on the first vector.

Thus, the component of velocity vr along the radial line is calculated by taking the magnitude of v multiplied by the projection of the unit vector of the radial line on v.

The component vr can be used to determine the speed of the car from the radar gun. The speed of the car is equal to the magnitude of vr divided by the speed of light.

By knowing the speed of the car, the speed limit can be compared to it in order to determine if the car is driving at a legal speed.

to know more about vector refer here:

https://brainly.com/question/24256726#

#SPJ11

what fraction of the engine power is being used to make the airplane climb? (the remainder is used to overcome the effects of air resistance and of inefficiencies in the propeller and engine.)

Answers

The fraction of the engine power used to make the airplane climb is 83% to two significant figures.

The power generated by the engine is given by the equation P=W/t, where P is power, W is work, and t is time.

The work done in lifting the airplane can be calculated as W=mgh, where m is mass, g is the gravitational constant, and h is the altitude gained.

Therefore, the power used to make the airplane climb can be calculated as P=mgh/t, where t is the time taken to gain the altitude.

Since the rate of climb is given as 2.5 m/s, the time taken to gain the altitude can be calculated as t=h/2.5, where h is the altitude gained.

Substituting the values into the equation, the power used to make the airplane climb can be calculated as P=750*9.8*h/2.5, where h is the altitude gained.

Therefore, the fraction of the engine power used to make the airplane climb is

P/(80*1000)=750*9.8*h/(2.5*80*1000).

Finally, the fraction of the engine power used to make the airplane climb expressed as a percentage is

(750*9.8 h/(2.5*80*1000))*100=83%.

For more question on engine click on

https://brainly.com/question/28206778

#SPJ11

if the force of friction opposing the motion is 18 n, what force f (in n) is the person exerting on the mower? (enter the magnitude.)

Answers

The force f (in n) the person is exerting on the mower is 18 n.

The force of friction opposing the motion refers to the force that acts in the opposite direction to the motion of an object and makes it harder to move.

According to the given statement, the force of friction opposing the motion is equal to the force f (in N) the person is exerting on the mower.

This indicates that the person's pushing force must be equal to the force of friction opposing the motion for the mower to move at a constant speed.

Using the above information, we can calculate the force f (in N) that the person is exerting on the mower as 18 n since it is equal to the force of friction opposing the motion.

Therefore, the person must push with a force f (in N) of 18 n to overcome the friction and maintain a constant speed for the mower.

To know more about  force of friction click here:

https://brainly.com/question/30280752

#SPJ11

g a research rocket is launched from boulder straight towards the south. how would the coriolis effect change the path of the rocket?

Answers

For a rocket launched southward from Boulder, the Coriolis effect would cause it to drift to the east, leading to a curved flight path rather than a straight one.

The Coriolis effect is an important force to consider when launching a research rocket from Boulder. The Coriolis effect is the result of Earth's rotation and will cause any object moving along the surface of the Earth to veer to the right in the Northern hemisphere and to the left in the Southern hemisphere.

This effect is most noticeable for objects traveling long distances, such as a rocket. As it continues to fly south, the Coriolis force will continue to act upon it, increasing the curvature of its path. The magnitude of the Coriolis force depends on the speed of the object and its distance from the poles. Therefore, the more time the rocket has to travel, the more it will be deflected from its intended path.

The Coriolis effect is an important factor to consider for any research rocket launch. It has the potential to affect the accuracy and success of the mission and must be taken into account when planning a launch trajectory.

For more such questions on Coriolis effect.

https://brainly.com/question/14290551#

#SPJ11

Complete Question:

A research rocket is launched from Boulder straight towards the south. How would the Coriolis effect change the path of the rocket?

a light has the frequency of 4.74 x 1014 sec-1 (hz). what is the wavelength? please show all the steps and all of your work when you upload your final answer.

Answers

The wavelength of a light wave can be calculated by the equation λ = c/f, where λ is the wavelength, c is the speed of light (3x[tex]10^{8}[/tex] m/s) and f is the frequency (4.74 x [tex]10^{14}[/tex] [tex]sec^{-1}[/tex]). Therefore, the wavelength of the light wave is 6.32 x [tex]10^{-7}[/tex] m.


The given frequency is 4.74 x 1014 sec-1. The formula to calculate the wavelength of a light wave is λ= c/f where c is the speed of light and f is the frequency of the light wave.

Therefore, λ= c/f= (3.00 x [tex]10^{8}[/tex] m/s)/(4.74 x [tex]10^{14}[/tex] [tex]sec^{-1}[/tex])= 6.32 x [tex]10^{-7}[/tex] m or 632 nm (rounding to three significant figures).

The wavelength of light is 6.32 x [tex]10^{-7}[/tex] m or 632 nm (rounding to three significant figures).

Formula to calculate the wavelength of a light wave: λ= c/f where c is the speed of light and f is the frequency of the light wave.

For more such questions on Wavelength.

https://brainly.com/question/3075536#

#SPJ11

joshua sees two different colored stars in the night sky. based on his observations, what can joshua infer about the two stars?

Answers

Based on Joshua's observation that he sees two different colored stars in the night sky, he can infer that the two stars have different temperatures.

When Joshua sees two different colored stars in the night sky, he can infer that the two stars have different temperatures. This is because the colors of stars depend on their temperatures. When a star is blue, it means that it's hotter than a star that is yellow or red.

As a result, Joshua can infer that the two stars have different temperatures due to their colors.A star's temperature is determined by its color. The color of a star is determined by its surface temperature.

Read more about stars :

https://brainly.com/question/30318208

#SPJ11

how much heat is lost through a 3' x 5' single-pane window with a storm that is exposed to a temperature differentia

Answers

The amount of heat lost through a 3' x 5' single-pane window with a storm that is exposed to a temperature differential is 108 BTU per hour.

The U-factor is a measure of how well a window insulates against heat transfer. The lower the U-factor, the better the window insulates.

The temperature difference is the difference between the inside and outside temperatures.The area of the window is the size of the window.

Using these factors, we can calculate the rate of heat loss through the window in units of BTUs per hour.

Assuming a U-factor of 1.2 and a temperature difference of 60°F, the calculation would be:

Heat Loss = 1.2 BTU/(hrft^2F) x 15 ft^2 x 60°F

Heat Loss = 108 BTU/hour

Therefore, the heat lost through the window is 108 BTU per hour.

To know more about heat loss click here:

https://brainly.com/question/14228650

#SPJ11

Complete Question:

How much heat is lost through a 3' × 5' single-pane window with a storm that is exposed to a 60 Fahrenheit temperature differential?

which of the choices below correctly lists things in order from largest to smallest? a) local group, solar system, milky way, universe b) universe, milky way, local group, solar system c) solar system, local group, universe, milky way d) universe, local group, milky way, solar system e) milky way, universe, solar system, local group

Answers

The choices below correctly lists things in order from largest to smallest, is D) universe, local group, milky way, solar system.

The universe is the largest structure in existence, followed by the local group of galaxies (which contains our Milky Way), followed by the milky way itself, and finally the solar system which is the smallest structure.

In terms of scale, the universe is far larger than any other structure, containing billions of galaxies and stars within it. The local group is a cluster of galaxies containing around 54 galaxies, of which our milky way is one. The milky way is itself a large collection of stars and planets, including our solar system which contains the planets and moons of our own solar system. So, in summary, the correct ordering from largest to smallest is Universe, Local Group, Milky Way, Solar System.

Learn more about solar system at:

https://brainly.com/question/3453959

#SPJ11

Question about my assignment

Answers

Answer: A - Gold Foil Experiment

Explanation:

Answer:

A

Explanation: Rutherford's gold foil experiment.

what is the value of the acting force between the coils if current is 30 a, separation between the coils is 5 cm, and the radius is 50 cm

Answers

The value of the acting force between the two coils is approximately 5.65 N.

F = (μ₀/4π) * (2I₁I₂*l)/d

Substituting these values into the method, we get:

F = (4π × [tex]10^{-7}[/tex] T·m/A) * (230 A30 A*π m)/(0.05 m)

Simplifying the expression, we get:

F ≈ 5.65 N

Force is an agent that can change the state of motion or shape of an object. it is a vector amount that has both value and path. Force can be applied through direct contact or from a distance, such as through gravitational or electromagnetic fields. Pressure is measured in gadgets of newtons (N) inside the international gadget of units (SI). Some common examples of forces include friction, tension, gravity, and electromagnetic forces.

According to Newton's laws of motion, force is directly proportional to the rate of change of momentum of an object. This means that a larger force will cause a greater acceleration of an object, and a smaller force will cause a smaller acceleration. Understanding the concept of force is essential to many areas of physics, including mechanics, thermodynamics, and electromagnetism.

To learn more about Force visit here:  

brainly.com/question/30526425

#SPJ4

suppose a woman does 350 j of work and 9250 j of heat is transferred from her into the environment in the process.(a) What is the decrease in her internal energy, assuming no change in temperature or consumption of food? (That is, there is no other energy transfer.)(b) What is her efficiency?

Answers

(a) The decrease in internal energy of the woman, assuming no change in temperature or consumption of food is -9600 J (negative because energy is lost) and (b) her efficiency is 3.64%.

The woman does 350 J of work and 9250 J of heat is transferred from her into the environment in the process. Since the energy transferred as heat is not positive, it is not useful energy. It is energy that is not doing any work. Therefore, the total energy transferred from the woman is 9250 J (as heat).

(a) The decrease in internal energy of the woman, assuming no change in temperature or consumption of food is the sum of the energy transferred as heat and the energy used to do work.

[tex]\Delta U=Q-W[/tex]

where ΔU is the change in internal energy, Q is the heat added to the system, and W is the work done by the system. Since no heat is added to the system,

[tex]\Delta U=-W = -350 \ J - 9250\  J = -9600 \ J[/tex] (negative because energy is lost).

(b) The efficiency of a machine is defined as the ratio of useful work done by the machine to the total energy input. In this case, the woman is the machine.

Efficiency = Useful work output / Total energy input

Total energy input = energy transferred as heat + energy used to do work [tex]= 9250 \ J + 350 \ J = 9600 \ J[/tex]

Useful work output = Work done = 350 J

Therefore, the efficiency of the woman is

Efficiency = Useful work output / Total energy input

Efficiency [tex]= 350\  J / 9600\  J\times 100 = 0.0364\times 100 = 3.64%[/tex].

Learn more about efficiency:

https://brainly.com/question/3617034

#SPJ11

what was the ratio of a weight to its just noticeable difference weight when they were lifted what was the ratio of a weight to its just noticeable difference weight when the weight were placed in the subject's hands?

Answers

According to Weber's Law, the ratio of a weight to its just noticeable difference weight when placed in the subject's hands is 1 : 40.

The ratio of a weight to its just noticeable difference weight when it is lifted by a subject is 1 : 40. This implies that if the weight of an object is x, the minimum additional weight that can be added to it and be noticed by a subject is x/40.The ratio of a weight to its just noticeable difference weight when the weight is placed in the subject's hands is 1:20.

This implies that if the weight of an object is x, the minimum additional weight that can be added to it and be noticed by a subject when it is placed in their hands is x/20. The Weber-Fechner Law applies in this scenario. It is a relationship between the intensity of a stimulus and its perceived strength that states that the sensation is proportional to the logarithm of the stimulus' intensity.

Learn more about Weber's Law at https://brainly.com/question/19021235

#SPJ11

a fragment of a current-carrying wire has a cross-sectional area that increases as shown. 1) if the current that flows through the wire is uniform, where is the drift velocity the greatest?

Answers

According to the given statement, if the current that flows through the wire is uniform, the drift velocity is the greatest at the section of wire with diameter d.

As the current is uniform throughout the wire, so the current through a given cross-sectional area is the same. Also, the current density, J is given by:

J = I/A

where I is the current and A is the cross-sectional area of the wire. Thus, if the area of the cross-section of the wire is more, the current density will be less. The current density is inversely proportional to the area of the wire, i.e. J ∝ 1/A. Hence, the drift velocity is inversely proportional to the current density, i.e. v[tex]_d[/tex] ∝ 1/J.

Thus, the drift velocity is greater where the cross-sectional area is less. So, the drift velocity is greater at the section of wire with diameter d.

So, the answer is at the section of wire with diameter d

Learn more about drift velocity at https://brainly.com/question/4269562

#SPJ11

a photon in the rest frame of a star has a wavelength of 780 nm. an observer on earth measures it to be at a wavelength of 768 nm. the star is then:

Answers

The star is moving away from the observer. An observer on the Earth measured the wavelength of a photon as 768 nm, while the photon's rest wavelength was 780 nm.

What is the Doppler effect?

According to the Doppler effect, if the wavelength of the wave is measured at different positions, it will shift. In this situation, the observer on the Earth is seeing a shift in the photon's wavelength due to the motion of the star.

To be more specific, when the star moves away from the observer, the observer observes an increase in the wavelength. Therefore, the star is moving away from the observer.

To know more about the Doppler effect:

https://brainly.com/question/5871604

#SPJ11

Other Questions
a cross of 2 sweet pea plants with white flowers results in f1 plants with wild type purpleflowers. purple flowers are dominant and white flowers are recessive. how many genes are involved in this cross? Why might it be helpful for an organism to have different types of connective tissue? the idea that individuals and organizations are accountable to a larger society is known as group of answer choices QUESTION 3 3.1 Define the term fraud and cite TWO examples of fraud that would have taken place within a company like Tencent. (1+2) (3) 3.2 Explain the concept embezzlement. 3.3 Differentiate between nepotism and bribery. 3.4 Explain TWO ways in which corruption impacts on a company's image. (1 x 2) (2) (2 x 2) (4) (2 x 2) (4) 3.5 What do you think are the causes of fraud and corruption within a workplace? (2 x 2) (4) 3.6 Research and advice business employers on what they can do to prevent employees from committing fraud at work. (2 x 2) (4) 3.7 Assess THREE measures the Government has put in place to deal with corruption in society. (3 x 2) (6) 3.8 Motivate why committing fraud or corruption is bad for the advance 10/10 ... zachary has purchased an investment that he expects to produce income of $3,000 at the end of the first year and $4,000 at the end of the second year. if he pays $5,800 for this investment, what is the internal rate of return? write a composition in french about your headmaster Katya is going on a 7-night trip. She is staying in a hotel that costs $129 per night and her airfare is $375. If she budged $2.000 for the trip, how much money will she have left for the trip?A. $672B. $686C.$704D.$722 if a firm has fixed costs of $20,000, variable cost per unit of $.50, and a breakeven point of 5,000 units, the price is: Helppp.. also show work pls Austin Airlines paid an annual dividend of $1.18 a share last month. The company is planning on paying $2, $3, and $3.5 a share over the next 3 years, respectively. After that, dividends will be growing at 3% per year indefinitely. What is the market price of this stock if the market rate of return is 10 percent? 1. The orthocenter is outside the triangle. 2. An altitude is the same line segment as an angle bisector. 3. If the perpendicular bisector of one side of a triangle intersects the opposite vertex, then the triangle is isosceles The graph represents a relation where x represents the independent variable and y represents the dependent variable. a graph with points plotted at negative 5 comma 1, at negative 2 comma 0, at negative 2 comma negative 2, at 0 comma 2, at 1 comma 3, and at 5 comma 1 What is the domain of the relation? {5, 2, 0, 1, 2, 3, 5} {5, 2, 1, 0, 3, 5} {5, 2, 0, 1, 5} {2, 0, 1, 3} The following amounts were taken from the financial statements of Thompson Corporation: 2018 2017 Total assets $800,000 $1,000,000 Net sales 760,000 720,000 Gross profit 352,000 320,000 Net Income 144,000 117,000 Weighted average number of 380,000 380,000 common shares Market price of common shares 44 48 The return on assets ratio for 2018 is a client signs a discretionary account agreement, his registered representative deciedes to buy some stock for the client on margin. the registered representative should 4. if the electric field of an electromagnetic wave is oscillating along the z-axis and the magnetic field is oscillating along the x-axis, in what possible direction is the wave traveling? true or false: the influential book the protocols of the elders of zion (1903) is a factual, historically accurate assessment of the jewish plan for world domination. which muscles are part of the rotator cuff muscles and what is their main function as a whole glaucoma can result from select one: a. a decrease in the number of cones. b. damage to the suspensory ligament. c. increased amounts of vitreous humor. d. inhibition of the circulation of aqueous humor. e. opacity of the lens. which type of phagocytic disorder occurs when white blood cells cannot initiate an inflammatory response to infectious organisms? 3. The program can potentiallyeven send drones to spray asubstance that can slow thespread of fire?